Quantum Queries on the Harmonic Oscillator

In summary, the conversation is about calculating the product of position and momentum uncertainty for a simple harmonic oscillator, and there is a mistake in the calculation of <\widehat{p}^{2}>. After correcting the mistake, the result is consistent with Heisenberg's uncertainty relation.
  • #1
T-7
64
0
Hi folks,

I wonder if I could run a few things past the quantum gurus among you - I'm just not quite convinced of some of the results I've been deriving.

Homework Statement



Consider the ground state of the simple harmonic oscillator at t = 0 with the normalised wave function [tex]\sqrt[4]{\frac{m\omega}{\pi\hbar}}e^{-\frac{m\omega}{2\hbar} x^2}[/tex].

If the position uncertainty is defined as [tex]\Delta x = \sqrt{<\widehat{x}^{2}>-<\widehat{x}>^{2}}[/tex] and the momentum uncertainty as [tex]\Delta p = \sqrt{<\widehat{p}^{2}>-<\widehat{p}>^{2}}[/tex], determine the product [tex]\Delta x . \Delta p[/tex]. Comment on the result.

The Attempt at a Solution



I begin by observing that [tex]<\widehat{x}>[/tex] and [tex]<\widehat{p}>[/tex] are both zero, by symmetry arguments. Thus [tex]\Delta x = \sqrt{<\widehat{x}^{2}>}[/tex] and [tex]\Delta p = \sqrt{<\widehat{p}^{2}>}[/tex].

I go on to calculate [tex]<\widehat{x}^{2}>[/tex] and [tex]<\widehat{p}^{2}>[/tex].

I find that [tex]<\widehat{x}^{2}> = \frac{\hbar}{2m\omega}[/tex], a result I am pretty confident of (it was an easy integration).

My queries arise for [tex]<\widehat{p}^{2}>[/tex], which comes out as [tex]\hbar(m\omega - \frac{1}{2})[/tex], giving [tex]\Delta x . \Delta p = \sqrt{\frac{\hbar}{2m\omega}} . \sqrt{\hbar(m\omega - \frac{1}{2})} = \hbar\sqrt{1/2 - 1/4m\omega}[/tex] - not a very "nice" result. I'm suspicious.

I make [tex]<\widehat{p}^{2}>[/tex] to be

[tex]<\widehat{p}^{2}> = \sqrt{\frac{m\omega}{\pi\hbar}}\int^{+\infty}_{-\infty} e^{-\frac{m\omega}{2\hbar} x^2}.(-\hbar^{2}\frac{\partial^2}{\partial x^2}).(e^{-\frac{m\omega}{2\hbar} x^2}) dx = . . . = \sqrt{m^{3}\omega^{3}\hbar/\pi}.I_{2} - \sqrt{m^{3}\omega^{3}/\pi\hbar}.I_{1}[/tex]

as

[tex](\frac{\partial^2}{\partial x^2}).(e^{-\frac{m\omega}{2\hbar} x^2}) = ... = -\frac{m\omega}{\hbar}.e^{-m\omega x^2/2\hbar} + (\frac{m\omega}{\hbar})^2.x^2.e^{-m\omega x^2/2\hbar}[/tex],

where

[tex]I_{1} = \int^{+\infty}_{-\infty}x^2.e^{-m\omega x^2/\hbar} dx = ... = \sqrt{\frac{\pi\hbar^{3}}{4m^{3}\omega^{3}}}[/tex]
[tex]I_{2} = \int^{+\infty}_{-\infty}e^{-m\omega x^2/\hbar} dx = ... = \sqrt{\frac{\pi\hbar}{m\omega}}[/tex]

etc.

determining finally that [tex]\hbar(m\omega - \frac{1}{2})[/tex]

Has something gone wrong here, perhaps, or should I just accept the result?

Cheers!
 
Last edited:
Physics news on Phys.org
  • #2
T-7 said:
Hi folks,

I wonder if I could run a few things past the quantum gurus among you - I'm just not quite convinced of some of the results I've been deriving.

Homework Statement



Consider the ground state of the simple harmonic oscillator at t = 0 with the normalised wave function [tex]\sqrt[4]{\frac{m\omega}{\pi\hbar}}e^{-\frac{m\omega}{2\hbar} x^2}[/tex].

If the position uncertainty is defined as [tex]\Delta x = \sqrt{<\widehat{x}^{2}>-<\widehat{x}>^{2}}[/tex] and the momentum uncertainty as [tex]\Delta p = \sqrt{<\widehat{p}^{2}>-<\widehat{p}>^{2}}[/tex], determine the product [tex]\Delta x . \Delta p[/tex]. Comment on the result.

The Attempt at a Solution



I begin by observing that [tex]<\widehat{x}>[/tex] and [tex]<\widehat{p}>[/tex] are both zero, by symmetry arguments. Thus [tex]\Delta x = \sqrt{<\widehat{x}^{2}>}[/tex] and [tex]\Delta p = \sqrt{<\widehat{p}^{2}>}[/tex].

I go on to calculate [tex]<\widehat{x}^{2}>[/tex] and [tex]<\widehat{p}^{2}>[/tex].

I find that [tex]<\widehat{x}^{2}> = \frac{\hbar}{2m\omega}[/tex], a result I am pretty confident of (it was an easy integration).

My queries arise for [tex]<\widehat{p}^{2}>[/tex], which comes out as [tex]\hbar(m\omega - \frac{1}{2})[/tex], giving [tex]\Delta x . \Delta p = \sqrt{\frac{\hbar}{2m\omega}} . \sqrt{\hbar(m\omega - \frac{1}{2})} = \hbar\sqrt{1/2 - 1/4m\omega}[/tex] - not a very "nice" result. I'm suspicious.

I make [tex]<\widehat{p}^{2}>[/tex] to be

[tex]<\widehat{p}^{2}> = \sqrt{\frac{m\omega}{\pi\hbar}}\int^{+\infty}_{-\infty} e^{-\frac{m\omega}{2\hbar} x^2}.(-\hbar^{2}\frac{\partial^2}{\partial x^2}).(e^{-\frac{m\omega}{2\hbar} x^2}) dx = . . . = \sqrt{m^{3}\omega^{3}\hbar/\pi}.I_{2} - \sqrt{m^{3}\omega^{3}/\pi\hbar}.I_{1}[/tex]
there is something wrong here. Look below at your second derivative of x^2 times the exponential. The two terms you get from the second derivative have a relative factor of m omega/ hbar. and yet in the above expression th erelative factor is not m omega/hbar. Check that equation.

as

[tex](\frac{\partial^2}{\partial x^2}).(e^{-\frac{m\omega}{2\hbar} x^2}) = ... = -\frac{m\omega}{\hbar}.e^{-m\omega x^2/2\hbar} + (\frac{m\omega}{\hbar})^2.x^2.e^{-m\omega x^2/2\hbar}[/tex],

where

[tex]I_{1} = \int^{+\infty}_{-\infty}x^2.e^{-m\omega x^2/\hbar} dx = ... = \sqrt{\frac{\pi\hbar^{3}}{4m^{3}\omega^{3}}}[/tex]
[tex]I_{2} = \int^{+\infty}_{-\infty}e^{-m\omega x^2/\hbar} dx = ... = \sqrt{\frac{\pi\hbar}{m\omega}}[/tex]

etc.

determining finally that [tex]\hbar(m\omega - \frac{1}{2})[/tex]

Has something gone wrong here, perhaps, or should I just accept the result?

Cheers!
 
  • #3
nrqed said:
there is something wrong here. Look below at your second derivative of x^2 times the exponential. The two terms you get from the second derivative have a relative factor of m omega/ hbar. and yet in the above expression th erelative factor is not m omega/hbar. Check that equation.

You are right. The expression should have read:

[tex]<\widehat{p}^{2}> = \sqrt{\frac{m\omega}{\pi\hbar}}\int^{+\infty}_{-\infty} e^{-\frac{m\omega}{2\hbar} x^2}.(-\hbar^{2}\frac{\partial^2}{\partial x^2}).(e^{-\frac{m\omega}{2\hbar} x^2}) dx = . . . = \sqrt{m^{3}\omega^{3}\hbar/\pi}.I_{2} - \sqrt{m^{5}\omega^{5}/\pi\hbar}.I_{1}[/tex]

This eventually leads to [tex]\hbar m\omega / 2[/tex], and then we do indeed derive Heisenberg's uncertainty relation in the form [tex]\hbar / 2 [/tex]

Many thanks.
 

1. What is a quantum query on the harmonic oscillator?

A quantum query on the harmonic oscillator is a mathematical calculation that is used to describe the behavior of a quantum system, specifically the harmonic oscillator. It involves solving the Schrodinger equation for the system to determine its energy levels and corresponding wave functions.

2. How is a quantum query on the harmonic oscillator different from a classical query?

A quantum query on the harmonic oscillator takes into account the principles of quantum mechanics, such as superposition and uncertainty, which differ from classical mechanics. This leads to different energy levels and wave functions for the system compared to a classical query.

3. What are the applications of quantum queries on the harmonic oscillator?

Quantum queries on the harmonic oscillator have various applications in physics and engineering. They can be used to model the behavior of atoms, molecules, and other quantum systems. They are also used in quantum computing for solving certain computational problems.

4. Can quantum queries on the harmonic oscillator be solved analytically?

In general, quantum queries on the harmonic oscillator cannot be solved analytically for arbitrary potentials. However, for certain specific potentials, such as the simple harmonic oscillator potential, analytical solutions can be obtained.

5. How do quantum queries on the harmonic oscillator contribute to our understanding of quantum mechanics?

Quantum queries on the harmonic oscillator provide a framework for understanding the behavior of quantum systems and how they differ from classical systems. They also help us make predictions about the behavior of these systems and provide insight into the principles of quantum mechanics.

Similar threads

Replies
1
Views
1K
  • Advanced Physics Homework Help
Replies
1
Views
2K
  • Advanced Physics Homework Help
Replies
10
Views
555
Replies
10
Views
316
  • Advanced Physics Homework Help
Replies
8
Views
925
  • Advanced Physics Homework Help
Replies
24
Views
783
  • Advanced Physics Homework Help
Replies
3
Views
880
Replies
2
Views
1K
Replies
3
Views
786
Back
Top